5
$\begingroup$

Suppose $\Gamma$ is the fundamental group of a closed, oriented surface $S$. Let $B$ be a finitely generated, infinite index subgroup of $\Gamma$, and let $\Gamma_B$ be the compact core of the $B$-covering space of $S$ (well-defined up to isotopy in the $B$-covering space). Lifting hyperbolic structures gives a well-defined continuous function from $T(\Gamma)$, the Teichmuller space of hyperbolic structures $\Gamma$, to $T(\Gamma_B)$, the Teichmuller space of hyperbolic structures with totally geodesic boundary on $\Gamma_B$.

The general question I would like to ask is: How rigid is the map $T(\Gamma) \to T(\Gamma_B)$? However, as it stands that question suffers from the problem that the boundary components of $T(\Gamma_B)$ can be arbitrarily long. So, in order to sweep that problem under the rug, I want to compose with the map from $T(\Gamma_B)$ to the Masur-Minsky marking complex $M(\Gamma_B)$, defined in their paper "Geometry of the curve complex II".

Question 1: How rigid is the composed map $T(\Gamma) \to T(\Gamma_B) \to M(\Gamma_B)$?

EDIT: What I mean by "rigidity" here is, loosely speaking, some kind of measurement of the extent of the image of this map in the marking complex $M(\Gamma_B)$. Here are two examples which exhibit opposite extremes of rigidity.

For example, if $B$ is the fundamental group of an essential subsurface of $\Gamma$ then the composed map is essentially surjective. This is the "least rigid" possibility.

For another example, if $B$ has rank $2$ and $\Gamma_B$ is a pair of pants then the composed map has bounded image in $M(\Gamma_B)$. This is the "most rigid" possibility (although this example is uninteresting because $M(\Gamma_B)$ is itself just a single point).

Question 2: Are there examples of subgroups $B$ where $\Gamma_B$ is not just a pair of pants but the image of the composed map $T(\Gamma) \to M(\Gamma_B)$ has finite diameter?

As a reminder of how the map $T(\Gamma_B) \to M(\Gamma_B)$ is defined, one fixes an appropriate Margulis constant $\epsilon$ for $\Gamma_B$, chosen so that for any hyperbolic structure in $T(\Gamma_B)$ the collection of closed geodesics and closed proper arcs of length $\le \epsilon$ fill $\Gamma_B$, and from that collection one uses surgery to construct a pants decomposition of $\Gamma_B$ and a transverse closed curve for each pants curve.

This question occurred to me while pondering a question of Benjamin Steinberg. I am not aware of any literature on this question.

It would also make sense to ask this question with regard to a natural "lifting" map $M(\Gamma) \to M(\Gamma_B)$, but the image of this map is probably coarsely the same as the image of the map $T(\Gamma) \to M(\Gamma_B)$.

$\endgroup$
7
  • $\begingroup$ @Lee: It will never be bounded (except for the pair of pants example): Just apply powers your favorite pseudo-Anosov to a non-peripheral element of $B$. Could you clarify what do you mean by rigidity in Question 1? The setup sounds somewhat similar to McMullen's work on theta-operator (or, even more remotely, skinning map). The restriction map $T(\Gamma)\to T(\Gamma_B)$ is holomorphic, so it contracts Teichmuller metric. It might follow from McMullen's work that the map is a strict contraction. $\endgroup$
    – Misha
    May 3, 2012 at 18:05
  • $\begingroup$ I edited to try to clarify what "rigid" means, although it is still a somewhat vague notion. $\endgroup$
    – Lee Mosher
    May 3, 2012 at 18:11
  • $\begingroup$ @Misha: just because the lengths of all elements of $B$ are uniformly long does not mean that the image is unbounded in the marking complex $M(\Gamma_B)$. For example, $\Gamma_B$ could be a one-holed torus with three very short properly embedded arcs hitting the boundary at right angles, cutting the torus into two right-angled hexagons. On this $\Gamma_B$, one could force the boundary length off to infinity without changing which three arcs are short, and the marking would be unchanged. $\endgroup$
    – Lee Mosher
    May 3, 2012 at 18:20
  • $\begingroup$ added: also, as the boundary length goes off to infinity, all lengths of all closed curves would have a lower bound that goes off to infinity. $\endgroup$
    – Lee Mosher
    May 3, 2012 at 18:21
  • $\begingroup$ @Lee: If $B$ contains a simple non-peripheral loop $\beta$ which projects to a non-simple loop in $S$ then the restriction map $f$ will always miss the thin part of $T(\Gamma_B)$ corresponding to $\beta$. Thus, the map is never coarsely surjective unless $B$ is a subsurface or a pair of pants. Also, for dimension reasons, the image is "too small" if $-\chi(B)> -\chi(S)$. An interesting question then would be if $f$ is (sometimes/always) coarsely onto thick part of $T(\Gamma_B)$ when $-\chi(B)\le -\chi(S)$ and $B$ is not embedded. "Sometimes" is probably true, "always" is probably false. $\endgroup$
    – Misha
    May 3, 2012 at 18:28

1 Answer 1

2
$\begingroup$

Regarding Question 2, you get lots of examples that are rigid for the lifting map $M(\Gamma) \to M(\Gamma_B)$.

Let $B$ be finitely generated subgroup of $\Gamma$ (considered a fuchsian group) such that every nontrivial element of $B$ fills $S$ (there are lots of these). Proposition 5.1 of [Kent, Leininger, and Schleimer. Trees and mapping class groups. J. Reine Angew. Math., 637:1–21, 2009] states that if you pull back all the essential simple closed curves on $S$ to the cover corresponding to $B$ and intersect with the convex core $\mathrm{core}(B)$, you get a finite set of arcs up to proper isotopy.

The idea of the proof is this: Suppose that there are arcs obtained by pulling back simple closed curves to $\mathrm{core}(B)$ whose lengths tend to infinity. These arcs limit (after passing to a subsequence) to a lamination $\widetilde \lambda$ on $\mathrm{core}(B)$ that covers a lamination $\lambda$ on $S$. But then there is a loop in $\mathrm{core}(B)$ that is not filling in $S$ (since the boundary of the supporting subsurface of $\widetilde \lambda$ will project to a curve in $S$ that misses $\lambda$). So there is a uniform bound to length of any of the arcs in our collection, and so the collection of isotopy classes is bounded.

So, if you lift any marking, you land in a finite set of markings for $\Gamma_B$.

It seems to me that, with some more work, this argument should give you a finite set of such arcs even if you allow $\Gamma$ to range over all of Teichmuller space (Dowdall and Leininger and I have been considering similar things, so I'll talk to them), which would answer Question 2 in its original form. Though it's probably easier to just say that the lifting map is essentially the same as $T(\Gamma) \to M(\Gamma_B)$, as you suggest.

$\endgroup$
3
  • $\begingroup$ Very nice! $\endgroup$
    – Lee Mosher
    May 6, 2012 at 23:24
  • 1
    $\begingroup$ Leininger has pointed out to me that the set of arcs does not depend on $\Gamma$, since they are determined by topological data (the lifts of simple curves on $S$ to $\mathbb{H}^2$ ``link" the limit set, and this linking data, which is really topological, determines the arcs you get). So the map $T(\Gamma) \to M(\Gamma_B)$ will be bounded for these filling groups $B$. $\endgroup$ May 8, 2012 at 20:12
  • 1
    $\begingroup$ Also, you should get other rigid examples by taking subgroups that are purely filling with respect to a subsurface. $\endgroup$ May 8, 2012 at 20:13

Your Answer

By clicking “Post Your Answer”, you agree to our terms of service and acknowledge you have read our privacy policy.

Not the answer you're looking for? Browse other questions tagged or ask your own question.